36th IMO 1995 shortlist

------
 
 
Problem N7

Does there exist n > 1 such that the set of positive integers may be partitioned into n non-empty subsets so that if we take an arbitrary element from every set but one then their sum belongs to the remaining set?

 

Solution

Answer: no.

Obviously no partition is possible for n = 2. So assume n > 2. Let the partition be A1 ∪ A2 ∪ ... ∪ An. We show first that if a, b, c (not necessarily distinct) belong to the same subset then a + b and a + c must belong to the same subset.

So suppose a, b, c belong to A1, a+b belongs to A2 and a+c belongs to A3. Take any elements a4 in A4, a5 in A5, ... , an in An. Then c + (a+b) + a4 + ... + an must be in A3. But it equals b + (a+c) + a4 + ... + an which must be in A2. Contradiction.

Suppose a+b belongs to A1 and a+c belongs to A3. As before take any ai in Ai. Then b + a3 + ... + an must be in A2, so (b + a3 + ... + an) + (a+c) + a4 + ... + an = (a+b+c) + (a3 + ... + an) + (a4 + ... + an) must be in A1. But c + a3 + ... + an must be in A2, so (a+b) + (c + a3 + ... + an) + a4 + ... + an = (a+b+c) + (a3 + ... + an) + (a4 + ... + an) must be in A3. Contradiction. The only remaining possibility is that a+b and a+c belong to the same subset, as claimed.

Now take ai in Ai. Put s = a1 + ... + an. Without loss of generality s is in A1. Put bi = s - ai. We can also write bi as a sum of an element from each subset except Ai, so bi must be in Ai. Now ai + bi = s, which is in A1. Hence ai + ai = 2ai is also in A1 (by the preliminary result).

Now suppose 2m in any even positive integer. We have m in some Ai. Take the set bj, where bj = aj except for bi = m. The same argument shows that 2b1, 2b2, ... , 2bn are all in the same subset. But if we take any j not equal to i, we already know that 2bj = 2aj is in A1. So 2m must also be in A1. In other words, all the even numbers are in A1.

Consider a1 + a3 + ... + an. It is in A2. Keep a3, a4, ... , an fixed and let a1 run through all the even numbers. Then we see that all odd numbers greater than a3 + ... + an must be in A2. Similarly, by considering a1 + a2 + a4 + ... + an, we see that all odd numbers greater than a2 + a4 + ... + an must be in A3. Contradiction.

 


 

36th IMO shortlist 1995

© John Scholes
jscholes@kalva.demon.co.uk
20 Sep 2002